Cours et vidéos

Cours en ligne Vidéos classées

Concours corrigés

HECS
HECE

Programme de concours

HECS

Chaîne Youtube


Pour me soutenir



Autour du site

Auteur du site.

Corrigé HEC math II 2010

Thèmes abordés

Remarque

Il s'agit d'un sujet technique et alambiqué. Idéal pour travailler les problèmes de convergences d'intégrales. Malheureusement le sujet manque à certains moment de clarté. Ainsi dans la partie II la phrase "$X_\alpha$ est indépendante de chacune des variables $(Y_k)_k$" pourrait laisser croire que les variables sont deux à deux indépendantes. Or ce n'est pas le cas, il faut en fait comprendre : "les variables aléatoires $X_\alpha,\ (Y_k)_{k}$ sont indépendantes."

Conseils

On prendra garde à bien vérifier les convergences des intégrales généralisées. On se souviendra aussi que l'espérance d'une variables aléatoires continue existe s'il y a convergence de $\int |x|f(x)dx$. On n'oubliera pas non plus d'invoquer systématiquement l'indépendance des variables aléatoires pour utiliser le produit de convolution (ça ne marche pas sinon).


Partie I


Dans tout le problème :

On rappelle ou on admet sans démonstration les résultats suivants :

L'objet du problème est de démontrer quelques propriétés de la fonction $\Gamma$ en utilisant des méthodes essentiellement probabilistes.

Partie I. Quelques résultats préliminaires

1. On pose pour tout n de $\mathbb N^*$ : $\displaystyle h_n=\sum_{k=1}^n\frac{1}{k}$. On considère les deux suites $(\gamma _n)_{n\geq 1}$ et $(v_n)_{n\geq 1}$ définies par : pour tout n de $\mathbb N^*$, $\gamma_n=h_n-\ln n$ et $v_n=\gamma_{n+1}-\gamma_n$.

a) Montrer que la série de terme général $v_n$ est convergente.

Afficher

Par calcul, on a : $$v_n=\gamma_{n+1}-\gamma_n=\frac{1}{n+1}-\ln\left(1+\frac{1}{n}\right)\overset{+\infty}{\sim}\frac{1}{n+1}-\frac{1}{n}\overset{+\infty}{\sim}-\frac{1}{n^2}.$$ Et on conclue facilement en utilisant le fait que $\sum\frac{1}{n^2}$ est une série de Riemann convergente.

b) En déduire la convergence de la suite $(\gamma_n)_{n\geq 1}$; on note $\gamma$ sa limite.

Afficher

$\sum_{k=1}^nv_n$ est une somme téléscopique et vaut $\gamma_{n+1}-\gamma_1$. Or la série $\sum v_n$ converge, donc $\gamma_n$ aussi.

c) On pose pour tout réel $t>0$ : $d_{n,t}=\gamma+\ln(t+n)-h_n$. Déterminer $\displaystyle\lim_{n\to\infty}d_{n,t}$.

Afficher

Un petit calcul donne $d_{n,t}=\gamma-\gamma_n+\ln\left(1+\frac{t}{n}\right)$ qui tend vers 0.

2.a) Rappeler sans démonstration les valeurs respectives de $E(X_t)$ et $V(X_t)$.

Afficher

Une loi gamma de paramètre t a pour espérance $\frac{1}{t}$ et pour variance $\frac{1}{t^2}$.

b) On note pour tout réel $t>0$, $\psi(t)=\Gamma'(t)/\Gamma(t)$, et $\psi'$ la dérivée de $\psi$. Montrer que $E(\ln(X_t))=\psi(t)$ et $V(\ln(X_t))=\psi'(t)$.

Afficher

Grâce à la formule de transfert, on a : $$E(\ln(X_t))=\frac{1}{\Gamma(t)}\int_{-\infty}^{+\infty}\ln(x)e^{-x}x^{t-1}dx=\frac{\Gamma'(t)}{\Gamma(t)}=\psi(t).$$ On calculera de même la variance à l'aide de la formule de transfert pour établir l'autre égalité.

3.a) Montrer que pour tout réel $t>1$, $E(1/X_t)$ existe et calculer sa valeur.

Afficher

Toujours grâce à la formule de transfert : $$E\left(\frac{1}{X_t}\right)=\frac{1}{\Gamma(t)}\int_0^{+\infty}e^{-x}x^{t-2}dx,$$ qui est une intégrale convergente pour $t>1$. D'autre part cette dernière intégrale vaut $\frac{\Gamma(t-1)}{\Gamma(t)}=\frac{\Gamma(t-1)}{(t-1)\Gamma(t-1)}=\frac{1}{t-1}$.

b) Etablir pour tout réel $x>0$, l'encadrement : $1-\frac{1}{x}\leq\ln x\leq x-1$.

Afficher

On étudiera par exemple les fonctions $x\mapsto \ln x-1+\frac{1}{x}$ et $x\mapsto \ln x-x+1$ à l'aide de tableaux de variations.

En déduire que l'on a : $(\ln x)^2\leq\left(1-\frac{1}{x}\right)^2+(x-1)^2$.

Afficher

On prendra garde que $\ln$ change de signe. Si $x\geq 1$, $\ln x\geq 0$ donc $(\ln x)^2\leq(x-1)^2\leq(x-1)^2+(\frac{1}{x}-1)^2$. Si maintenant $x\leq 1$, $0\leq-\ln x\leq\frac{1}{x}-1$ et $(\ln x)^2\leq(\frac{1}{x}-1)^2\leq(x-1)^2+(\frac{1}{x}-1)^2$.

c) A l'aide des question précédentes, établir les inégalités suivantes : pour tout réel $t>0$, $E\left(\ln\left(\frac{X_t}{t}\right)\right)\leq 0$; pour tout réel $t>1$, $E\left(\ln\left(\frac{X_t}{t}\right)\right)\geq -\frac{1}{t-1}$, pour tout réel $t>2$, $E\left(\ln^2\left(\frac{X_t}{t}\right)\right)\leq\frac{2t}{(t-2)^2}$.

Afficher

On sait que $\ln x\leq x-1$, donc par croissance et linéarité de l'espérance $$E\left(\ln\left(\frac{X_t}{t}\right)\right)\leq E\left(\frac{X_t}{x}-1\right)=\frac{1}{t}E(X_t)-1=\frac{1}{t^2}-1\leq 0.$$ Les deux autres inégalités s'obtiennent de la même façon en utilisant les inégalités établies en b.

d) Soit t un réel fixé strictement positif. Montrer que la suite de variables aléatoires $\left(\ln\left(\frac{X_{t+n}}{t+n}\right)\right)_{n\geq 1}$ converge en probabilité vers 0.

Afficher

On utilise l'inégalité de Markov : $$P\left(\left|\ln\left(\frac{X_{t+n}}{t+n}\right)\right|>\epsilon\right)=P\left(\left|\ln\left(\frac{X_{t+n}}{t+n}\right)\right|^2>\epsilon^2\right)\leq \frac{1}{\epsilon^2}E\left(\ln^2\left(\frac{X_{t+n}}{t+n}\right)\right)\frac{1}{\epsilon^2}\frac{2(t+n)}{(t+n-2)^2}\overset{+\infty}{\longrightarrow}0.$$ Il y a donc bien convergence en proba vers 0.

4. Soit $(A_n)_{n\geq 1}$, $(B_n)_{n\geq 1}$ et $(C_n)_{n\geq 1}$, trois suites de variables aléatoires à densité qui convergent en probabilité vers 0. On pose pour tout n de $\mathbb N^*$ : $D_n=A_n+B_n+C_n$. Soit $(u_n)_{n\geq 1}$ une suite réelle qui converge vers u. On considère deux variables aléatoires réelles à densité M et N telles que pour tout n de $\mathbb N^n$, M est de même loi que $N+D_n+u_n$.

a) Montrer que pour tout réel $\epsilon>0$, l'inclusion : $\left[|D_n|>\epsilon\right]\subset\left[|A_n|>\epsilon/3\right]\cup\left[|B_n|>\epsilon/3\right]\cup\left[|C_n|>\epsilon/3\right]$.

En déduire que la suite $(D_n)_{n\geq 1}$ converge en probabilité vers 0.

Afficher

On observe d'abord grâce à l'inégalité triangulaire que : $$|A_n|+|B_n|+|C_n|\geq|A_n+B_n+C_n|=|D_n|.$$ Donc $|D_n|>\epsilon\Rightarrow |A_n|+|B_n|+|C_n|>\epsilon$. D'autre part les trois termes ne peuvent pas tous être plus petit que $\epsilon/3$ sinon la somme serait inférieure ou égale à trois. Il suit que : $$|D_n|>\epsilon\Rightarrow |A_n|>\epsilon/3\text{ ou }|B_n|>\epsilon/3\text{ ou }|C_n|>\epsilon/3.$$ L'implication se traduisant en inclusion et le "ou" en union, on en déduit l'inclusion recherchée. Enfin grâce à la propriété $P(A\cup B)\leq P(A)+P(B)$ et la propriété $A\subset B\Rightarrow P(A)\leq P(B)$, on a que : $$P(|D_n|>\epsilon)\leq P(|A_n|>\epsilon/3)+P(|B_n|>\epsilon/3)+P(|C_n|>\epsilon/3),$$ Or chacun des trois termes de droite tend vers 0 par convergence en proba donc $D_n$ tend vers 0 en proba.

b) On pose pour tout n de $\mathbb N^*$ : $V_n=D_n+u_n-u$. Montrer que la suite de variables aléatoires $(V_n)_{n\geq 1}$ converge en probabilité vers 0. En déduire la limite en probabilité de la suite $((N+u)+V_n)_{n\geq 1}$.

Afficher

On peut raisonner comme dans la question précédente, c'est à dire on a l'inclusion suivante : $$\left[|V_n|>\epsilon\right]\subset\left[|D_n|>\epsilon\right]\cup\left[|u_n-u|>\epsilon\right].$$ Comme $u_n$ tend vers u, pour n suffisemment grand l'évènement $\left[|u_n-u|>\epsilon\right]$ est vide donc de proba nulle et on a : $$P(|V_n|>\epsilon)\leq P(|D_n|>\epsilon).$$ Enfin on conclut avec le fait qu'on a montré que $D_n$ tend vers 0 en proba. Enfin on a que : $$P\left(\left|(N+u+V_n)-(N+u)\right|>\epsilon\right)=P(|V_n|>\epsilon)\overset{+\infty}{\longrightarrow}0,$$ par convergence vers 0 en proba de $V_n$. Il suit que $N+u+V_n$ tend vers $N+u$ en proba.

c) On admet sans démonstration que la convergence en probabilité entraine la convergence en loi. Montrer que les variables aléatoires M et N+u sont de même loi.

Afficher

On a $N+u+V_n=N+D_n+u_n$ qui a même loi que $M$ et comme la convergence en proba implique la convergence en loi, le résultat découle de b.

5. Soit $(\alpha,\beta)$ un couple de réels strictement positifs, et $X_\alpha$ et $X_\beta$ deux variables indépendantes de lois respectives $\gamma_\alpha$ et $\gamma_\beta$. On pose : $T_{\alpha,\beta}=\frac{X\alpha}{X_\beta}$, $Q_{\alpha,\beta}=\ln(T_{\alpha,\beta})$ et $B(\alpha,\beta)=\frac{\Gamma(\alpha)\Gamma(\beta)}{\Gamma(\alpha+\beta)}$.

a) Préciser $Q_{\alpha,\beta}(\Omega)$. Déterminer une densité de $\ln(X_\alpha)$ et de $-\ln(X_\beta)$ respectivement.

Afficher

Comme une loi gamma prend ses valeurs dans $\mathbb R^+$, $T_{\alpha,\beta}$ prend ses valeurs dans $\mathbb R^+$. Enfin $\ln:\mathbb R^+\to\mathbb R$ donc $Q_{\alpha,\beta}(\Omega)=\mathbb R$. Pour une densité de $\ln X_\alpha$, on calcule d'abord la fonction de répartition : $$P(\ln X_\alpha<\!x)=P(X_\alpha<\! e^x)=F_{X_\alpha}(e^x),$$ puis on la dérive. On trouve alors : $$f_{\ln X_\alpha}(x)=\left(P(X_\alpha<\! e^x)\right)'=e^x\frac{1}{\Gamma(\alpha)}e^{e^{x}}e^{x\alpha-x}=\frac{e^{-e^x}e^{\alpha x}}{\Gamma(\alpha)}.$$ De même on a : $$P(-\ln X_\beta<\!x)=P(X_\beta> e^{-x})=1-F_{X_\beta}(x).$$ Puis en dérivant, on trouve : $$f_{-\ln X_\beta}(x)=\frac{e^{e^{-x}}e^{-\beta x}}{\Gamma(\beta)}.$$

b) En déduire qu'une densité $f_{Q_{\alpha,\beta}}$ de $Q_{\alpha,\beta}$ est donnée par : pour tout réel x, $$f_{Q_{\alpha,\beta}}(x)=\frac{e^{-\beta x}}{\Gamma(\alpha)\Gamma(\beta)}\int_{-\infty}^{+\infty}e^{(\alpha+\beta)y}\exp\left(-e^y(1+e^{-x})\right)dy.$$

Afficher

Il suffit de se souvenir que si X et Y sont deux variables indépendantes de densité $f_X$ et $f_Y$, alors la densité de $X+Y$ est le produit de convolution : $$f_{X+Y}(x)=\int_{-\infty}^{+\infty}f_X(x-t)f_Y(t)dt,$$ et de remarquer que $Q_{\alpha,\beta}=\ln X_\alpha+(-\ln X_\beta)$.

c) A l'aide du changement de variable $u=e^y(1+e^{-x})$, dont on justifiera la validiré, établir la formule suivante : pour tout x réel, $f_{Q_{\alpha,\beta}}=\frac{1}{B(\alpha,\beta)}\times\frac{e^{\alpha x}}{(1+e^x)^{\alpha+\beta}}$.

Afficher

On n'oubliera pas que pour les intégrales impropres, il faut s'assurer que le changement de variable est de classe $C^1$ strictement monotone. Ici l'application $y\mapsto e^{y}(1+e^{-x})$ est $C^1$ strictement croissante de $\mathbb R$ dans $\mathbb R^+$ donc les nouveaux bords de l'intégrale seront $\int_0^{+\infty}$. Après le changement de variable, c'est un petit calcul!

d) En déduire une densité $f_{T_{\alpha,\beta}}$ de $T_{\alpha,\beta}$.

Afficher

On raisonne encore avec les fonctions de répartitions en n'oubliant pas que $T_{\alpha,\beta}$ prend ses valeurs dans $\mathbb R^+$ et on trouve : $$f_{T_{\alpha,\beta}}(x)=\begin{cases}& 0\text{ si }x\leq 0\\ & \frac{x^{\alpha-1}}{B(\alpha,\beta)(1+x)^{\alpha+\beta}}\text{ si }x>0\end{cases}$$

e) On pose : $J_{\alpha,\beta}=\frac{X_\alpha}{X_\alpha+X_\beta}$. Montrer qu'une densité de $J_{\alpha,\beta}$ est donnée par : $$f_{\alpha,\beta}(z)=\begin{cases}& 0 \text{ si }z\not\in ]0,1[\\ & \frac{1}{B(\alpha,\beta)}z^{\alpha-1}(1-z)^{\beta-1} \text{ si } z\in ]0,1[\end{cases}.$$

Afficher

On observe d'abord que $J_{\alpha,\beta}$ est à valeurs dans $]0,1[$. Ensuite, calcule la fonction de répartion, pour tout $x\in]0,1[$ : $$P(J_{\alpha,\beta}\geq x)=P\left(\frac{X_\alpha}{X_\alpha+X_\beta}\leq x\right)=P\left(\frac{X_\alpha}{X\beta}\leq\frac{x}{1-x}\right)=F_{T_{\alpha,\beta}}\left(\frac{x}{1-x}\right),$$ puis on procède comme précédemment en dérivant.

Pour afficher le fil des commentaires : Commentaires.


Pour poster un commentaire ou obtenir de l'aide : c'est ici!




Formulaire

L'insertion de formules suit la syntaxe LATEX. Toute formule doit être encadrée par des dollars : $\bf{\$formule\$}$. Par exemple $\bf{\$ u\_n \$}$ sera interprétée comme une formule et donnera $\bf{u_n}$. Voici quelques exemples pour ceux qui ne sont pas habitués :

Contacter l'auteur du site : frederic.millet @ math-sup.fr